Is the Intersection of T-Invariant Subspaces Always T-Invariant?

  • Thread starter Thread starter something
  • Start date Start date
  • Tags Tags
    Subspaces
Click For Summary
The discussion centers on proving that the intersection of any collection of T-invariant subspaces is itself a T-invariant subspace. The initial proof correctly demonstrates this for two subspaces but fails to generalize for an arbitrary number of subspaces. The grader points out that the proof needs to explicitly account for an infinite or uncountable collection of T-invariant subspaces. To correct the proof, it should use a general index set to represent the subspaces instead of limiting it to just two. The conclusion is that the proof must be adapted to encompass all possible collections of T-invariant subspaces.
something
Messages
1
Reaction score
0

Homework Statement


Prove that the intersection of any collection of T-invariant subspaces of V is a T-invariant subspace of V.

Homework Equations


The Attempt at a Solution


Let W1 and W2 be T-invariant subspaces of V. Let W be their intersection.
If v\inW, then v\inW1 and v\inW2. Since v\inW1, T(v)\inW1 & v\inW2, T(v)\inW2. Therefore T(v)\inW.
For any x,y\inW, x,y\inW1 and x,y\inW2, x,y\inW and cx+y\inW since W1 and W2 are subspaces.
Thus the intersection of any collection of T-invariant subspaces is a T-invariant subspace.

My answer was marked wrong. The grader's comment was to cross out "any" and replace it with "2". What I should have said? Was I supposed to explicitly point out that this applies from 2...n? :(
 
Physics news on Phys.org
Your grader is right. Your proof shows that given two invariant spaces W1 and W2, that their intersection is invariant.
Your proof does not shows that given n invariant subspaces W1, W2,..., Wn, that their intersection is invariant.
 
In fact, the problem says "the intersection of any collection" so showing this for "n invariant subspaces" would not be sufficient. The collection does not have to be finite nor even countable.

Your proof will work nicely if you modify it so that instead of "W1" and "W2", you use W<sub>\alpha</sub> where \alpha is simply some label distinguishing the different sets- that is, \alpha is just from some index set, not necessarily {1, 2}, nor even a set of numbers.
 
Question: A clock's minute hand has length 4 and its hour hand has length 3. What is the distance between the tips at the moment when it is increasing most rapidly?(Putnam Exam Question) Answer: Making assumption that both the hands moves at constant angular velocities, the answer is ## \sqrt{7} .## But don't you think this assumption is somewhat doubtful and wrong?

Similar threads

  • · Replies 4 ·
Replies
4
Views
3K
  • · Replies 1 ·
Replies
1
Views
2K
  • · Replies 4 ·
Replies
4
Views
2K
Replies
12
Views
2K
  • · Replies 5 ·
Replies
5
Views
7K
  • · Replies 10 ·
Replies
10
Views
2K
  • · Replies 7 ·
Replies
7
Views
2K
  • · Replies 8 ·
Replies
8
Views
2K
  • · Replies 3 ·
Replies
3
Views
2K
  • · Replies 7 ·
Replies
7
Views
2K